0 Daumen
131 Aufrufe

Die Frage ist, ob dieses Integral endlich ist. Ich weiss nicht wie ich da vorgehe…

IMG_0656.jpeg

Text erkannt:

\( \int \limits_{0}^{\infty} \frac{x^{2}}{1+x^{2}+x^{4}} d x \)

Ich habe versucht jegliches zu substituieren, aber es brachte nichts. Wie kann man bei soetwas die Stammfunktion berechnen?

Avatar von

Hier wird es dir gezeigt:

https://www.integralrechner.de/

@simplemind

Der Integralrechner gibt dem FS vor, wie er das ausrechnen kann, aber das braucht der FS hier überhaupt gar nicht.

1 Antwort

0 Daumen
 
Beste Antwort

Hallo.

Die Stammfunktion interessiert dich hier gar nicht. Achte auf deine Aufgabenstellung! Du sollst hier nicht das Integral explizit bestimmen, sondern nur zeigen, das es endlich ist, also konvergiert.

Schätze die Funktion im Integral nach oben ab und finde eine Funktion, dessen Integral eine konvergente Majorante ist.

Tipp: Für jedes x ∈ |R gilt die Abschätzung:

x^2 / (1+ x^2 + x^4) ≤ 1/ (1+x^2).

Überlege dir mal wie diese Abschätzung zustande kommt und was du nun damit anfangen kannst.

Avatar von 1,3 k

Ah verstehe, weil man x^2 ausklammern kann und dann im Nenner der Kehrwert davon immer positiv ist, sodass man den für die obere Abschätzung weglassen kann. Und das Integral müsste dann der Arcustangens sein, wovon das Integral von 0 bis unendlich endlich ist. Richtig?

Richtig. Das uneigentliche Integral vom Arcustangens existiert hier, da die Grenzwerte lim (x —> inf) arctan(x) = π/2 und lim (x —> 0) arctan(x) = arctan(0) = π/4 existieren. Damit ist das eine konvergente Majorante und daraus folgt, das das Integral nach dem Vergleichstest endlich ist.

Super

danke :D

Ein anderes Problem?

Stell deine Frage

Willkommen bei der Mathelounge! Stell deine Frage einfach und kostenlos

x
Made by a lovely community